Showing that an element is a unit in a ring

  • Thread starter Thread starter Mr Davis 97
  • Start date Start date
  • Tags Tags
    Element Ring Unit
Click For Summary
To show that an element ##\alpha## in the ring of integral Hamiltonian quaternions is a unit if ##N(\alpha) = 1##, it is established that ##N(\alpha) = \alpha\bar{\alpha}##. Given that ##\alpha\bar{\alpha} = 1##, it follows that ##\bar{\alpha}## must be the multiplicative inverse of ##\alpha##. The discussion emphasizes the need to demonstrate that ##\bar{\alpha} \alpha = 1##, which is essential for confirming that ##\alpha## is indeed a unit in the ring. The relationship between the quaternion and its conjugate is crucial, as both must belong to the set ##S##. Ultimately, proving the inverse relationship solidifies ##\alpha##'s status as a unit.
Mr Davis 97
Messages
1,461
Reaction score
44

Homework Statement


Let ##S=\{a+bi+cj+dk \mid a,b,c,d \in \mathbb{Z}\}## be the ring of integral Hamiltonian quaternions, where multiplication is defined using the same rules as in ##\mathbb{H}##, the ring of real Hamiltonian quaternions. Define a function $$N:S\to\mathbb{Z}, N(a+bi+cj+dk)=a^2+b^2+c^2+d^2.$$

Show that if ##N(\alpha)=1##, then ##\alpha## is a unit

Homework Equations

The Attempt at a Solution


So suppose we know that ##N(\alpha) = \alpha\bar{\alpha}##, where the bar is the conjugate of the quaternion. If ##N(\alpha)=1##, then ##\alpha\bar{\alpha} = 1##. So to show that ##\alpha## is a unit, all I need to show is the other direction, that ##\bar{\alpha}\alpha=1##. How can I do this? In groups one implies the other since we have inverses, but this is not the case with rings.
 
Physics news on Phys.org
If you already know the inverse, show that it is in ##S## and that it actually is the inverse.
 
fresh_42 said:
If you already know the inverse, show that it is in ##S## and that it actually is the inverse.
Well all I know is that ##\alpha\bar{\alpha} = 1##. Clearly ##\bar{\alpha}\in S##, so all I need to show that ##\bar{\alpha}\alpha = 1##. But I don't see how to get that
 
Mr Davis 97 said:
Well all I know is that ##\alpha\bar{\alpha} = 1##. Clearly ##\bar{\alpha}\in S##, so all I need to show that ##\bar{\alpha}\alpha = 1##. But I don't see how to get that

If ##\alpha=a+bi+cj+dk## and you define ##\bar{\alpha}=a-bi-cj-dk## so that ##\alpha \bar \alpha=a^2+b^2+c^2+d^2##, doesn't that make ##\bar \alpha \alpha=a^2+(-b)^2+(-c)^2+(-d)^2##?
 
  • Like
Likes Mr Davis 97
Question: A clock's minute hand has length 4 and its hour hand has length 3. What is the distance between the tips at the moment when it is increasing most rapidly?(Putnam Exam Question) Answer: Making assumption that both the hands moves at constant angular velocities, the answer is ## \sqrt{7} .## But don't you think this assumption is somewhat doubtful and wrong?

Similar threads

  • · Replies 2 ·
Replies
2
Views
2K
Replies
1
Views
1K
Replies
4
Views
2K
Replies
2
Views
2K
Replies
9
Views
5K
  • · Replies 1 ·
Replies
1
Views
1K
Replies
1
Views
2K
Replies
7
Views
2K
  • · Replies 4 ·
Replies
4
Views
2K
  • · Replies 1 ·
Replies
1
Views
2K